3D geometry parallelepiped problem

Click For Summary
The discussion revolves around demonstrating that points K and L, where the diagonal AG intersects planes BDE and CFH, are the centers of gravity for those planes. The user outlines their approach, defining G1 and G2 as the centers of gravity for planes BDE and CFH, respectively. They show that G1 lies on the line AG and belongs to plane BDE, establishing K = G1, while G2 is similarly derived for plane CFH, establishing L = G2. The user notes that K and L are located at one-third and two-thirds along the diagonal AG. A visual representation of the parallelepiped was requested to aid in understanding.
geoffrey159
Messages
535
Reaction score
72

Homework Statement


[/B]
Given a rectangular parallelepiped ABCDEFGH, the diagonal [AG] crosses planes BDE and CFH in K and L. Show K and L are BDE's and CFH's centres of gravity.

I think I have understood the problem, could you verify my demo please ? Thanks

Homework Equations



The Attempt at a Solution



Let us call ##G_1## and ##G_2## BDE's and CFH's center of gravity.

## \vec{AG} = \vec{AB} + \vec{AD} + \vec{AE} = 3 \vec{AG_1}##

So ## A,G_1, G## are aligned and ##G_1## belongs to plane BDE so we must have ##K = G_1##

Then

## 3 \vec{AG_2} = \vec{AC} + \vec{AF} + \vec{AH} = \vec{AB} + \vec{BC} + \vec{AE}+\vec{EF} + \vec{AD} + \vec{DH} = \vec{AG} + \vec{BC} + \vec{EF}+\vec{DH} = 2\vec{AG}##

So, similarly, ## A,G_2, G## are aligned and ##G_2## belongs to plane CFH so we must have ##L = G_2##

Furthermore, ##K## and ##L## are at the third and two third of the diagonal.
 
Physics news on Phys.org
It would help if you provided a picture of your regular parallelepiped with vertices labeled.
 
Hi, here is the picture
 

Attachments

  • drawing.png
    drawing.png
    20.8 KB · Views: 322
Question: A clock's minute hand has length 4 and its hour hand has length 3. What is the distance between the tips at the moment when it is increasing most rapidly?(Putnam Exam Question) Answer: Making assumption that both the hands moves at constant angular velocities, the answer is ## \sqrt{7} .## But don't you think this assumption is somewhat doubtful and wrong?

Similar threads

  • · Replies 7 ·
Replies
7
Views
3K